(3 intermediate revisions by the same user not shown)
Line 3: Line 3:
 
<math>\lambda(B)=\frac{\mu(B)}{\mu(A)}</math>
 
<math>\lambda(B)=\frac{\mu(B)}{\mu(A)}</math>
  
This is clearly a measure on <math>A</math> with <math>\lambda(A)=1</math>
+
This is clearly a measure on <math>A</math> with <math>\lambda(A)=1 \frac{}{}</math>
  
Moreover, <math>\int_{A}fd\mu = \mu(A)\int_A f d\lambda \frac{}{}</math>
+
Moreover, <math>\int_{A}fd\mu = \mu(A)\int_A f d\lambda \frac{}{}</math>.
 +
 
 +
By Jensen's Inequality, we get
 +
 
 +
<math>\phi(\int_Afd\lambda) \leq \int_A\phi(f)d\lambda = \frac{\int_A\phi(f)d\mu}{\mu(A)}</math>.
 +
 
 +
So <math>\phi(\frac{\int_Afd\mu}{\mu(A)}) \leq \frac{\int_A\phi(f)d\mu}{\mu(A)}</math>

Latest revision as of 10:52, 22 July 2008

Define a function from the set of all measurable subset $ B $ of $ A $ as below

$ \lambda(B)=\frac{\mu(B)}{\mu(A)} $

This is clearly a measure on $ A $ with $ \lambda(A)=1 \frac{}{} $

Moreover, $ \int_{A}fd\mu = \mu(A)\int_A f d\lambda \frac{}{} $.

By Jensen's Inequality, we get

$ \phi(\int_Afd\lambda) \leq \int_A\phi(f)d\lambda = \frac{\int_A\phi(f)d\mu}{\mu(A)} $.

So $ \phi(\frac{\int_Afd\mu}{\mu(A)}) \leq \frac{\int_A\phi(f)d\mu}{\mu(A)} $

Alumni Liaison

ECE462 Survivor

Seraj Dosenbach